Contracts - Perspectives

Ace your homework & exams now with Quizwiz!

What happens if the term is to be agreed on lateR?

ften, an offer will state that some term is to be agreed on at a future date. If the term is a material term, the offer is too uncertain. The courts will not supply a reasonable term, as the parties have provided otherwise. However, as discussed above, Article 2 permits a reasonable price term to be supplied by the court under these circumstances if the other evidence indicates that the parties intended to form a contract.

Silence as acceptance?

general rule: offeree should not be legally obligated to affirmatively reject an offer but if you accept benefits thats acceptance. this is especially true if prior dealing between the parties or trade practices known to both create a commercialy reasonable expectation by the offeror that silence represents an acceptance.

What happens when the breach is by the builder?

1. Breach before construction started - Owner gets cost of completion PLUS reasonable compensation for any delay in performance. 2. if breach during construction - If the builder breaches after partially performing, the owner is entitled to the cost of completion plus reasonable compensation for any delay in performance.If, however, completion would involve undue economic waste, the measure of damages will be the difference between the value of what the owner would have received if the builder had properly performed the contract and the value of what the owner actually received. example: Homeowner and Builder enter into a contract to build a house. The contract provides, among other things, that all of the plumbing pipes will be copper. After the plumbing is installed throughout the house, but before construction of the house is completed, Homeowner discovers that the pipes installed were made of polyvinyl chloride ("PVC"), and not copper. Homeowner insists that Builder remove the entire plumbing system and replace the PVC pipes with copper pipes. The house with PVC pipes would be valued at $500 less than it would have been had copper pipes been installed. However, it would cost Builder $10,000 in labor and materials to rip out the PVC pipes and replace them with copperipes. Result: Builder would not be compelled to replace the pipes, and Homeowner's damages would be $500, which may be offset against the amount owed to Builder. 3. if builder breaches by late performance - If the builder completes performance, but it is late, the owner has a right to damages for any loss incurred by not being able to use the property when perfor- mance was due, e.g., loss of reasonable rental value when property could have been leased. However, if damages for this "lost use" are not easily determined or were not foreseeable at the time the contract was entered into, the owner can recover only the interest on the value of the building as a capital investment.

What are the exceptions to the perfect tender rule?

1. Installment contracts - The right to reject when a contract is an installment contract (i.e., when there is to be more than one delivery) is much more limited than in a single delivery contract situa- tion. Installment contracts follow a rule akin to the common law substantial perfor- mance doctrine. In an installment contract situation, an installment can be rejected only if the nonconformity substantially impairs the value of that installment and cannot be cured (see below). In addition, the whole contract is breached only if the nonconformity substantially impairs the value of the entire contract. example: Steve and Becky enter into a contract under which Steve is to deliverto Becky 100 blue widgets on the first day of each month, and Beckyis to pay Steve $275 by the 10th of each month. Steve makes a perfect delivery the first two months and Becky makes the required payments. On the first day of the third month, Steve sends only 90 widgets. The 10-widget shortfall would be a basis for rejection under the perfect tender rule, but because this is an installment contract, Becky cannot reject the installment unless she can show that the 10-widget shortfall substantially impairs the value of that installment, and she cannot cancel the entire contract unless she can show that the shortfall substantially impairs the value of the entire contract.

What responsibility does the buyer have for goods after rejection?

1. uyer Must Hold Goods with Reasonable Care After rejecting goods in her physical possession, the buyer has an obligation to hold them with reasonable care at the seller's disposition for a time sufficient to permitthe seller to remove them. If the seller has no agent or place of business within the market area where the goods are rejected, a merchant buyer has an obligation to obey any reasonable instructions as to the rejected goods (i.e., she must arrange to reship the goods to a destination designated by the seller or resell on request of the seller, if reasonable). 2. If a seller gives no instructions within a reasonable time after notification of rejection, the buyer may reship the goods to the seller, store them for the seller's account, or resell them for the seller's account. The buyer has a security interest in rejected goods in her possession for any part of the price already paid and for expenses reasonably incurred in connection with handling them after rejection. 3. If the buyer does resell rejected goods, she is entitled to have her expenses of selling and any commission ordinarily paid in the trade or, if there is none, a reasonable commis- sion not exceeding 10%.

What is a commercial unit?

A "commercial unit" is one that by commercial usage is treated as a single whole for the purpose of sale, and division of which materially impairs its value (e.g., place setting of dishes). A commercial unit may be a single article (e.g., a machine) or a set of articles (e.g., a suite of furniture), a quantity (e.g., a bale, a gross), or any other unit treated in use or in the relevant market as a single whole. [UCC §2-105(6)] The test for "commercial unit" is "not only what unit has been the basis of contract, but also whether the partial acceptance produces so materially an adverse effect upon the remainder as to constitute bad faith."

is a buyer's right to reject cut off by acceptance?

A buyer's right to reject under the perfect tender doctrine generally is cut off by acceptance. Under Article 2, a buyer accepts when: 1. After a reasonable opportunity to inspect the goods, she indicates to the seller that they conform to requirements or that she will keep them even though they fail to conform; 2. She fails to reject within a reasonable time after tender or delivery of the goods or fails to seasonably notify the seller of her rejection; or 3. She does any act inconsistent with the seller's ownership

What are the two ways buyer can get damages?

Difference Between Contract Price and Market Price OR Difference between contract price and cost of replacement goods.

What happens when owner breaches in construction contract?

Breach by Owner If the owner has breached, check to see when the breach occurred. 1. IF BREACH BEFORE CONSTRUCTED STARTED BUILDER GETS PROFITS HE WOULD HAVE DERIVED from K 2. If the breach occurs during construction, the builder is entitled to any profit he would have derived from the contract plus any costs he has incurred to date. The formula is also stated as the contract price minus the cost of completion. 3. Breach after construction completed: If the breach occurs after construction has been completed, the builder is entitled to the full contract price plus interest thereon.

Construction contract damages?

Check to see whether the owner or builder is breaching.

Employment contracts damages?

Check to see who did the breach employer or employee Breach by Employer Irrespective of when the breach occurs—i.e., before performance, after part perfor- mance, or after full performance, the standard measure of the employee's damages is the full contract price. Breach by Employee If an employee materially breaches an employment contract, the employer is entitled to recover the cost of replacing the employee (i.e., the wages the employer must pay to a replacement employee minus the breaching employee's wages). The breaching employee may offset money owed for work done to date.

What are incidental damages?

Compensatory damages may also include incidental damages. Incidental damages are most commonly associated with contracts for the sale of goods and typically include expenses reasonably incurred by a buyer in inspection, receipt, transporta- tion, care, and custody of goods rightfully rejected and other expenses reasonably incident to the seller's breach, and by the seller in storing, shipping, returning, and reselling the goods as a result of the buyer's breach.

What are consequential damages?

Consequential damages are special damages and reflect losses over and above standard expectation damages. These damages result from the nonbreaching party's particular circumstances. Usually, consequential damages are lost profits resulting from the breach. These damages may be recovered only if at the time the contract was made, a reasonable person would have foreseen the damages as a probable result of a breach. Foreseeability is the key issue for consequential damages. To recover consequential damages, the plaintiff must show that the breaching party knew or had reason to know of the special circumstances giving rise to the damages. example: Alex and Becky make a written contract under which Alex is to recondition by a stated date a used machine owned by Becky so that it will be suitable for sale by Becky to Cindy. Alex knows when they make the contract that Becky has contracted to sellthe machine to Cindy but knows nothing of the terms of Becky's contract with Cindy. Because Alex delays in returning the machine to Becky, Becky is unable to sell it to Cindy and loses the profit that she would have made on that sale. Becky's loss of reasonable profit was foreseeable by Alex as a probable result of the breach at the time the contract was made. compare: Suppose in the above example that the profit that Becky would have made was extraordinarily large because Cindy promised to pay an exceptionally high price as a result of a special need forthe machine. Again Alex is aware of the contract with Cindy but unaware of its terms. Alex is not liable for Becky's loss of profit beyond what would ordinarily result from such a contract. The exceptionally high price paid by Cindy was not foreseeable by Alex as a probable result of the breach at the time the contract was made.

What happens when the buyer gets the difference between contract price and cost of replacement goods?

Cover is the usual measure of damages for a buyer. Typically, if a buyer isnot sent the goods contracted for, he will go out into the marketplace to buy replacement goods. If the buyer chooses the cover measure (i.e., difference between contract price and cost of buying replacement goods), the buyer must make a reasonable contract for substitute goods in good faith and without unreasonable delay. example: Seller and Buyer have a contract for the sale of 10,000 widgets at $1 per widget. Seller does not deliver. At the time and place for determining market price, the average price of widgets is $1.05. However, Buyer made a replacement contract within a reasonable time and in good faith at a price of $1.07. Buyer can recover $700 based on her replacement costs. If, on the other hand, Buyer could have bought substitute widgets for $1.03 while the general market price was $1.05, but she chose not to cover, she could recover $500 based on the difference between contract and market prices, rather than being limited to her cover costs.

Other than the penalty rule, what is the other requirement for liquidated damages?

Damages for contractual breach must have been difficult to estimate or ascertain at the time the contract was formed. The amount agreed on must have been a reasonable forecast of compensatory damages in the case of breach. The test for reasonableness is a comparison between the amount of damages prospectively probable at the time of contract formation and the liquidated damages figure. If the liquidated damages amount is unreasonable, the courts will construe this as a penalty and will not enforce the provision.

What are the two ways seller can get damages?

Difference between contract price and market price - market price measured as of the time and at the place for delivery Difference between contract price and resale price - This is the usual measure of a seller's damages. If the seller chooses to resell, he must do so under the provisions of section 2-706, which requires a good faith, commercially reasonable sale that may be either private or public (auction). In the case of a private sale, the breaching buyer must be given reasonable notice of intention to resell. In the case of an auction sale, the sale must be at a usual market for such goods if such a market is reasonably avail- able. Notice of the sale must be given to the breaching buyer unless the goods are perishable or threaten to decline rapidly in value. Only existing and identi- fied goods may be sold, unless there is a market in futures for the particular goods. The seller may buy the goods at an auction sale. For seller its contract price - market or resale

What are the buyer's nonmonetary remedies?

If a buyer rightfully rejects goods because they do not conform to the contract, one of her options is simply to cancel the contract. Buyer has right to replevy identified goods On buyer's inability to cover? - In addition, the buyer may replevy undelivered, identified goods from the seller if the buyer, after reasonable effort, is unable to secure adequate substitute good

What is the measure of damages for restitution?

Generally, the measure of restitution is the value of the benefit conferred. Although thisis usually based on the benefit received by the defendant (e.g., the increase in value of the defendant's property or the value of the goods received), recovery may also be measured by the "detriment" suffered by the plaintiff (e.g., the reasonable value of the work performed or the services rendered) if the benefits are difficult to measure or the "benefit" measure would achieve an unfair result. example: Homeowner hires Painter to paint Homeowner's house in exchange for $5,000. When the job is 70% complete, Homeowner orders Painter to stop because Homeowner does not like the new color. The value of the work performed thus far is $4,000. Although the value of Homeowner's house might not have been increased by the partially finished paint job, Painter may recover in a restitutionary action the $4,000 value of the services rendered.

what happens when a contract calls for installment payments and there is a breach?

If a contract calls for payments in installments and a payment is not made, there is only a partial breach. The aggrieved party is limited to recovering only the missed payment, not the entire contract price. However, the contract may include an acceleration clause making the entire amount due on any late payment, in which case the aggrieved party may recover the entire amount.

What is the notice part of the perfect tender rule?

If in connection with rejection the buyer fails to state that the goods have a particular defect that is ascertainable by reasonable inspection, she cannot rely on that defect to justify rejection or to show seller's breach if: 1. The seller could have cured the defect if he had been told about it; or 2. Between merchants when the seller has, after rejection, made a request in writing for a full and final written statement of all defects upon which the buyer proposes to rely. example: Buyer has ordered blue widgets. Buyer rejects because the shipment did not contain the widget wrench that, under the contract, went with each widget. Buyer does not give the reason for rejection. If Seller had known the reason, he could have had the necessary number of widget wrenches at Buyer's business within hours. That probably would have constituted an adequate cure. If so, Buyer's rejection is unjustified; she will not be able to rely on the absence of the wrenches as a reason for rejection or as the basis for a claim for damages.

What is the action for price rule?

If the buyer has accepted the goods and has not paid, or has not accepted the goods, and the seller is unable to resell them at any reasonable price, or if the goods have been lost or damaged at a time the risk of loss was on the buyer, the seller may maintain an action against the buyer for the full contract price.

What happens when buyer gets difference between contract price and market price?

If the buyer measures damages by the difference between contract price and market price, market price usually is determined as of the time the buyer learns of the breach and at the place of tender. [UCC §2-713] Note that the buyer's damages are measured as of the time she learns of the breach, while the seller's damages are measured as of the time for delivery.

What is specific performance?

If the legal remedy is inadequate, the nonbreaching party may seek specific performance, which is essentially an order from the court to the breaching party to perform or face contempt of court charges. The legal remedy (damages) generally is inadequate when the subject matter of the contract is rare or unique. The rationale is that if the subject matter is rare or unique, damages will not put the nonbreaching party in as good a position as perfor- mance would have, because even with the damages the nonbreaching party would not be able to purchase substitute performance. Usually available for land or rare paintings.

What is the reliance damage measure?

If the plaintiff's expectation damages will be too speculative to measure (e.g., the plaintiff cannot show with sufficient certainty the profits she would have madeif the defendant had performed the contract), the plaintiff may elect to recover those damages she has suffered based on her reasonable reliance on the contract. Reliance damages award the plaintiff the cost of her performance; i.e., they are designed to put the plaintiff in the position she would have been in had the contract never been formed. example: J-Mart gives Sam a "dealer franchise" to sell J-Mart's products in a stated area for one year. In preparation for performance, Sam spends money on advertising, hiring sales personnel, and acquiring premises that cannot be used for other purposes. J-Mart then repudiates before performance begins. If it cannot be established with reasonable certainty what profit Sam would have made if the contract had been performed (i.e., Sam's expectation damages), Sam can recover as reliance damages his expenditures in prepara- tion for performance.

Misunderstanding/Ambiguity Rule?

Neither Party Aware of Ambiguity—No Contract Both Parties Aware of Ambiguity—No Contract. One Party Aware of Ambiguity—Contract

Whats the two types of damages buyer gets?

Market or cover . COVER IS THE USUAL ONE.

What are the grounds for reformation?

Mistake To reform a contract because of mistake, there must be: (i) an agreement between the parties, (ii) an agreement to put the agreement in writing, and (iii) a variance between the original agreement and the writing. Misrepresentation - If a writing is inaccurate because of a misrepresentation, the plaintiff can choose between reformation and avoidance. To qualify for reformation, the misrepresentation must relate to the content or the legal effect of the record. The court will reform the writing to reflect the expressed intent of the parties. Misrepresentations as to the subject matter of the agreement are not grounds for reformation because the court will not remake the parties' bargain. Rescission and damages are the proper remedy for that. example: Seller owns Blackacre, which is encumbered by a mortgage. Sellerand Buyer agree that Buyer will purchase Blackacre subject to the mortgage. Buyer does not agree to assume the mortgage. If Seller inserts a clause under which Buyer agrees to assume the mortgage and Buyer signs without knowledge of this, Buyer is entitled to reformation.

How are contracts accepted under common law and article two, how are they formed under common law and article 2?

Offers can be accepted by promising or beginning performance Under common law for a bilateral contract the contract is formed if the person begins performance under common law a unilateral contract can only be formed if performance is complete. Starting performance does create an option though.

.Does the buyer have a right to revoke acceptance?

Once goods are accepted, the buyer's power to reject the goods generally is terminated and the buyer is obligated to pay the price less any damages resulting from the seller's breach. However, under limited situations, a buyer may revoke an acceptance already made. A proper revocation of acceptance has the effect of a rejection.

Does a seller have a right to cure beyond the original contract time?

Ordinarily, the seller has no right to cure beyond the original contract time. However, in cases where the buyer rejects a tender of nonconforming goods that the seller reasonably believed would be acceptable "with or without money allowance," the seller, upon a reasonable notification to the buyer, has a further reasonable time beyond the original contract time within which to make a conforming tender. A seller will probably be found to have had reasonable cause to believe that the tender would be acceptable if the seller can show that (i) trade practices or prior dealings with the buyer led the seller to believe that the goods would be acceptable, or (ii) the seller could not have known of the defect despite proper business conduct (e.g., packaged goods purchased from a supplier). example: 1) In the last example above, widgets are delivered without wrenches on June 15. Seller and Buyer have had a numberof contracts over the years for the sale of widgets in which the wrench was a part of the contract. On several occasions, Seller has not been able to deliver the wrenches, and on each occasion, Buyer has accepted the widgets with a reduction in price and purchased the wrenches from another source. This time Buyer rejects the widgets. Seller will have a reasonable time after June 15 within which to cure by furnishing the wrenches. 2. 2) Barry ordered 100 barrels of grade A oil from Sonya tobe delivered on or before January 1. On January 1, Sonya delivered to Barry 100 barrels of oil that she had purchased from her supplier, Refineco. Upon delivery, Barry opened a barrel and found that the oil was grade B oil. Barry immedi- ately rejected the delivery. Sonya checked with Refineco and discovered that Refineco had made a packaging error and could replace the oil within two days. Assuming two days is a reasonable time under the circumstances (e.g., if Barry does not need the oil immediately), Sonya will have a right to cure even though the time for performance has passed.

What is reformation?

Reformation is the remedy whereby the writing setting forth the agreement between the parties is changed so that it conforms to the original intent of the parties.

What is recission?

Rescission is a remedy whereby the original contract is considered voidable and rescinded. The parties are left as though a contract had never been made.

What is restitution?

Restitution is not really part of contract law, but rather is a distinct concept. Restitution is based on preventing unjust enrichment when one has conferred a benefit on another without gratuitous intent.

Restitution when no contract involved - Quasi contract remedy -

Restitution may also be available in a quasi-contract action when there is no contrac- tual relationship between the parties if: (i) The plaintiff has conferred a benefit on the defendant by rendering services or expending properties; (ii) The plaintiff conferred the benefit with the reasonable expectation of being compensated for its value; (iii) The defendant knew or had reason to know of the plaintiff's expectation; and (iv) The defendant would be unjustly enriched if he were allowed to retain the benefit without compensating the plaintiff. example: Doctor witnesses an automobile accident and rushes to aid an uncon- scious victim. Doctor can recover the reasonable value of his services.

Restitution when a contract unforeceable - Quasi contract rememdy?

Restitution may be available in a quasi-contract action when a contract was made but is unenforceable and unjust enrichment otherwise would result. examples: 1) Aristotle hires Derek to sign autographs in Aristotle's sporting goods store one day next month and gives Derek half of his $1,000 fee upon making the contract. Derek then dies and so is discharged from his obligation to perform. Aristotle can recover the $500 from Derek's estate as restitution in quasi-contract. 2) Owner hires Contractor to repair Owner's house. After Contractor has completed half of the repair work, the house is destroyed by a tornado. Although the parties will be discharged for impossibility, Contractor will be able to recover in restitution for the valuable improvements made to the house before it was destroyed. 3) Landlord promises to sell Tenant five acres of a 1,000-acre tract that Tenant is leasing, but the contract fails to state which five acres. Tenant plants fruit trees on the five acres that he thinks were intended. Tenant cannot enforce the promise because it does not specify which five acres were intended, but he can recover restitution in a quasi-contract action for the value of the fruit trees.

What is the seller's right to cure?

Seller Can Cure by Notice and New Tender Within Time for Performance - If the buyer has rejected goods because of defects, the seller may within the time originally provided for performance "cure" by giving reasonable notice of her intention to do so and making a new tender of conforming goods which the buyer must then accept. example: Buyer ordered blue widgets for delivery during the first 15 days of June. The widgets are delivered on June 9, but the widget wrenches required by the contract are missing. Seller can cure this defect by giving reasonable notice of his inten- tion to provide and subsequently providing wrenches for the widgets by June 15. If he does, Buyer must accept, or Buyer will breach the contract.

What is the lost volume seller example?

Seller, a distributor of widgets, can get all of the widgets he needs for sale. He makes a contract to sell 10,000 widgetsto Buyer at a price of $1 per widget. Buyer repudiates the contract. Seller resells the widgets he had identified to Buyer's contract to Z for $1 per widget. If damages are measured by the difference between the contract price and resale price, Seller will be denied recovery. However, assuming Seller paid 85¢ per widget for these widgets, his lost profit on the Buyer deal is $1,500 ($10,000 less $8,500), because even if Buyer had not breached, Seller would have been able to supply Z with widgets. Because Buyer's breach did not enable Seller to make the sale to Z, and because the sale to Z would have been made in any event, the only way to make Seller whole is to allow him to recover his lost profits, i.e., $1,500. If Seller would have incurred sales commissions of $500 and delivery expenses of $100 if Buyer had taken the goods, but does not now incur those expenses, the saved expenses reduce the recovery. Therefore, the recovery would be $900 ($1,500 less saved expenses of $600). Compare: Seller and Buyer enter into a contract for the sale of a particular painting by van Gogh at a price of $25,000. Seller paid $15,000 for the painting two years earlier. Buyer repudiates. Seller resells to Z at $24,000. Seller's measure of damages is $1,000 plus incidental damages. Seller cannot get the $10,000 lost profit measure (i.e., the difference between the contract price and what Seller paid for the painting) because there is only one painting, and Seller could not have made the sale to Z but for Buyer's repudiation.

What is the UCC liquid damages rule?

The UCC allows a court to consider actual damages to validate a liquidated damages clause. Even if the clause was not a reasonable forecast of damages at the time of the contract formation, it will be valid if it was reasonable in light of the subsequent actual damages.

When may acceptance be revoked in perfect tender rule situation?

The buyer may revoke her acceptance of goods if the goods have a defect that substan- tially impairs their value to her and: (1) She accepted them on the reasonable belief that the defect would be cured and it has not been; or She accepted them because of the difficulty of discovering defects or because of the seller's assurance that the goods conformed to the contract.

What are the buyer's basic damages?

The buyer's basic damages where the seller does not deliver or the buyer properly rejects or revokes her acceptance of tendered goods consist of the difference between the contract price and either the market price or the cost of buying replacement goods (i.e., cover), plus incidental and consequential damages (see above), if any, less expenses saved as a result of the seller's breach. so for buyer its contract price - either replacement or market

Lost Volume Seller Rule

The formula - lost profit is measured by the contract price with the breaching buyer minus cost to the seller. i if seller would have incurred losses but did not cuz buyer ****ed up contract then you gotta minus that from whatever the seller was gonna get back The previous two measures of damages might not give adequate compen- sation for the buyer's breach in situations where the seller can obtain or manufacture as many goods as he can sell (e.g., a car dealership). In such a case, the seller is known as a lost volume seller, because although he is able to resell the goods for the same or similar price as in the initial contract, he loses volume of business: But for the buyer's breach, the seller would have made two sales instead of one. Generally, lost profit is measured by the contract price with the breaching buyer minus cost to the seller.

Grounds for recission?

The grounds for rescission must have occurred either before or at the time the contract was entered into. The grounds are: 1. Mutual mistake of a material fact; 2. Unilateral mistake if the other party knew or should have known of the mistake; 3. Misrepresentation of fact or law by either party as to a material factor in the negotia- tions that was relied upon; and Other grounds, such as duress, undue influence, illegality, lack of capacity, and failure of consideration.

what is the certainty rule?

The plaintiff must prove that the losses suffered were certain in their nature and not speculative. Traditionally, if the breaching party prevented the nonbreaching party from setting up a new business, courts would not award lost profits from the prospective business as damages, because they were too speculative. However, modern courts may allow lost profits as damages if they can be made more certain by observing similar businesses in the area or other businesses previously owned by the same party.

What are compensatory damages?

The purpose of contract damages is to give compensation for the breach—i.e., to put the nonbreaching party in the position she would have been in had the promise been performed so far as money can do this. The most common measure of this isthe value of the breaching party's performance that was lost (expectation damages), plus incidental and consequential damages, less any loss or cost saved by not having to perform.

What are the seller's damages when buyer refuses to accept goods or anticipatory breaches contract

The seller's basic damages when the buyer refuses to accept goods or repudiates are either the difference between the contract price and the market price or the difference between the contract price and the resale price of the particular goods, plus incidental (but not consequential) damages, if any, less expenses saved as a result of the breach. If damages based on the difference between the contract price and market or resale price do not put the seller in as good a position as perfor- mance would have, then the seller may recover lost profits plus incidental damages. [UCC §§2-706, 2-708, 2-710] In the case of a buyer's anticipatory breach, the seller's damages are measured as of the actual time for performance, unless the suit comes to trial before the time for performance, in which case damages are measured as the time the seller learned of the breac

Damages for contracts for sale of land?

The standard measure of damages for breach of land sale contracts is the difference between the contract price and the fair market value of the land.

Does a buyer have a right to demand assurances?

Under Article 2, actions or circumstances that increase the risk of nonperformance by a party to the contract but do not clearly indicate that performance will not be forth- coming, may not be treated immediately as an anticipatory repudiation. Instead, if there are reasonable grounds for insecurity with respect to the other party's performance, a party may demand assurances that the performance will be forthcoming at the proper time. The demand for assurances must be made in writing. Until the party receives adequate assurances, he may suspend his own performance. [UCC §2-609] If the proper assurances are not given within a reasonable time (i.e., within 30 days after a justified demand for assurances), he may then treat the contract as repudiated. What constitutes an adequate assurance depends on the facts of the case. example: `. 1) Seller hears a rumor, in fact false, that Buyer is in financial trouble. Seller reasonably believes that the rumor may have foundation in fact. He is justified in making a demand for assurances and withholding any goods for which he has not been paid. Buyer, within a reasonable time, sends a financial report from her banker showing good financial condi- tion. This is adequate assurance and Seller must resume performance. 2. 2) Same facts as above except that Buyer is in bad financial condition. Adequate assurance may require a third party of good credit to back up Buyer. 3. 3) Same facts as above. Buyer does not give any assurances. Seller may treat the failure to give assurances as a repudiation of the contract.

Mitigation damages -

Under the common law, the nonbreaching party cannot recover damages that could have been avoided with reasonable effort. Thus, she must refrain from piling up losses after she receives notice of the breach; she must not incur further expenditures or costs, and she must make reasonable efforts to cut down her losses by procuring a substitute performance at a fair price. Should she not do so, she will not be allowed to recover those damages that might have been avoided by such mitigation after the breach. Generally, a party may recover the expenses of mitigation. Note the following specific contract situations: Employment Contracts If the breaching employer can prove that a comparable job in the same locale was available, then contract damages against that breaching employer for lost wages will be reduced by the wages that the plaintiff would have received from that comparable job. Manufacturing Contracts Generally, in a contract to manufacture goods, if the person for whom the goods are being manufactured breaches, the manufacturer is under a duty to mitigate by not continuing work after the breach. However, if the facts are such that completion of the manufacturing project will decrease rather than increase damages, the manufacturer has a right to continue. example: Partly manufactured goods may be without value because they cannot be sold. The nonbreaching manufacturer may complete production and recover for his expenses in doing so, because finished goods usually can be resold, and the damages will be decreased as a result.

What is the restoration and economic waste rule?

Usually, when a building contract is not properly performed, the owner is entitled to the cost of fixing the defect. However, as noted in the pipes example above, unless there is special significance attached to use of a particular item (e.g., the owner is the CEO of the particular brand of copper pipe specified) and that significance is communicated to the builder, a court will not order a remedy that results in undue economic waste. Moreover, courts are split on the result when a party contracts to restore property and willfully refuses to do so because it is much more costly than any diminution in value of the property. example: Farmer and GasCo enter into a two-year contract that permits GasCoto explore Farmer's property and extract any natural gas it finds in exchange for a fixed sum and a promise to restore the land to its pre-exploration status upon completion of the two-year term. At theend of the term, GasCo determines that it will cost $200,000 to restore the property. Although it cannot be used for farming, the land has lost only $5,000 in value. Courts are split on whether GasCo must pay the $200,000 to restore the property or only the $5,000 loss in value. GasCo will argue that to pay the $200,000 is economic waste. Farmer will argue that it bargained for the restoration and GasCo will be unjustly enriched if it does not have to follow through—giving GasCo a $195,000 windfall at Farmer's expense.

is the seller liable for consequential damages?

a seller is liable for consequential damages arising from his breach if: (i) he had reason to know of the buyer's general or particular requirements,and (ii) the subsequent loss resulting from those needs could not reasonably be prevented by cover. Particular needs must be made known to the seller, but general requirements usually need not be. if the buyer is in the business of reselling the goods, the seller is deemed to have knowledge of the resale. IF a seller knows that the goods he provides are to be used in the manufacturing process he should know that his breach would cause a disruption in production leading to a loss of profits.

Can you reject goods in an installment contract? -

generally no, Article 2 provides that a defective shipment in an installment contract cannot be rejected if the defect can be cured. Ordinarily, defects in the particular goods themselves cannot be cured, so the buyer can reject them, but then might be required to accept substitute goods under the provisions discussed above. Note that a deficiency in quantity may be cured by an additional delivery, and a delivery of too much may be cured by acceptance or return of a part.

what are the other requirements of revocation?

must be within a reasonable time has to be before any substantial change in the goods occur

Seller's nonmonetary goods?

selelr can withold goods ight to Recover from Buyer on Buyer's Insolvency If a seller learns that a buyer has received delivery of goods on credit while insolvent, the seller may reclaim the goods upon demand made within 10 days after the buyer's receipt of the goods. However, the 10-day limitation doesnot apply if a misrepresentation of solvency has been made in writing to the particular seller within three months before delivery. Note that the seller's right to reclaim the goods is subject to the rights of a buyer in the ordinary course or any other good faith purchaser. [UCC §2-702]

What is the standard measure of damages - expectation damages?

the plaintiff's standard measure of damages will be based on an "expectation" measure, i.e., sufficient damages for her to buy a substitute perfor- mance. This is also known as "benefit of the bargain" damages.

When is a rejection effective?

when received

How does an infant affrim a contract?

An infant may affirm, i.e., choose to be bound by his contract, upon reaching majority. He affirms either expressly or by conduct, e.g., failing to disaffirm the contract within a reasonable time after reaching majority.

What if there is a misunderstanding/Ambiguity?

Contract language with at least two possible meanings leads to different results depending on the awareness of the parties. Most often there is no contract because there is no meeting of the minds.

Minor breach?

A breach of contract is minor if the obligee gains the substantial benefit of her bargain despite the obligor's defective performance. Examples would be insignificant delays in completing performance or small deficiencies in the quality or quantity of performance when precision is not critical. The effect of a minor (immaterial) breach is to provide a remedy for the immaterial breach to the aggrieved party. The aggrieved party is not relieved of her duty of performance under the contract.

What are the two two elements of consideration?

(i) there must be a bargained- for exchange between the parties; and (ii) that which is bargained for must be considered of legal value or, as it is traditionally stated, it must constitute a benefit to the promisor or a detriment to the promisee. At the present time, the detriment element is emphasized in determining whether an exchange contains legal value. example: Jeff promises to sell his used television to Kristen for $100 in exchange for Kristen's promise to pay $100. Both elements of consideration are found in this example. First, Jeff's promise was bargained for. Jeff's promise induced a detri- ment in the promisee, Kristen. Kristen's detriment induced Jeff to make the promise. Second, both parties suffered detriments. The detriment to Jeff was the transfer of ownership of the television, and the detriment to Kristen was the payment of $100 to Jeff.

6 factor criteria for materiality of breach?

1) Amount of Benefit Received Look to the extent to which the nonbreaching party will receive substantially the benefit she could have anticipated from full performance. The greater the extent, the less material the breach. 2) Adequacy of Damages Look to the extent to which the injured party may be adequately compensated in damages. The greater the extent, the less material the breach. 3) Extent of Part Performance Look to the extent the party failing to perform completely has already performed or made preparations to perform. The greater the extent, the less material the breach. 4. Hardship to Breaching Party Look to the extent of hardship on the breaching party should the contract be termi- nated. If a finding of materiality and termination of the contract would cause great hardship to the breaching party, the breach is less likely to be found to be material. 5. 5) Negligent or Willful Behavior Look to the extent of negligent or willful behavior of the party failing to perform. The greater the extent, the more material the breach. 6. Likelihood of Full Performance Look to the extent of likelihood the party who has failed to perform will perform the remainder of his contract. The greater the extent, the less material the breach.

does a statement that makes implicit terms explicit prevent acceptance?

1) Distinguish—Statements that Make Implicit Terms Explicit Statements by the offeree that make implicit terms explicit do not prevent acceptance. Example: The statement by an offeree, "I accept provided you convey market- able title," is a valid acceptance because the obligation to convey marketable title is implicit in the offer to sell.1) Distinguish—Statements that Make Implicit Terms Explicit Statements by the offeree that make implicit terms explicit do not prevent acceptance. Example: The statement by an offeree, "I accept provided you convey market- able title," is a valid acceptance because the obligation to convey marketable title is implicit in the offer to sell.

Preexisting Duty Rule exceptions?

1. Exception—New or Different Consideration Promised If the promisee has given something in addition to what she already owes in return 2. Exception—Voidable Obligation A promise to perform a voidable obligation (i.e., ratification) is enforceable despite the absence of new consideration. Thus, an infant's (i.e., minor's) ratification of a contract upon reaching the age of majority is enforceable without new consider- ation, as is a defrauded person's promise to go through with the tainted contract after learning of the fraud. 3. xception—Preexisting Duty Owed to Third Party Traditionally, when a preexisting duty was owed to a third party, courts held that the new promise did not constitute consideration. However, the modern view adopted by the Second Restatement and the majority of jurisdictions states that the new promise constitutes consideration. [See Restatement (Second) of Contracts §73]. example: Saul Pimon contracts with Pam Promotor to sing at a concert in New York for $25,000. Later, when Pimon threatens to cancel, Dud Dooright, a Pimon fan, offers to pay Pimon an additional $5,000 if he sings at the concert. Pimon appears and sings as agreed. Under the traditional view, Pimon cannot enforce Dooright's promise to pay the additional $5,000, but under the majority view Pimon can enforce the promise because Pimon did not owe a duty to Dooright under the original contract. 4. Exception—Honest Dispute as to Duty If the scope of the legal duty owed is the subject of honest dispute, then a modifying agreement relating to it will ordinarily be given effect. The compromise by each party is a detriment. 5. xception—Unforeseen Circumstances Under the modern view, which appears to be the view adopted by the National Conference of Bar Examiners for MBE purposes, a promise modifying a contract that has not been fully performed on either side is binding without consideration if the modification is fair and equitable in view of circumstances not anticipated when the contract was made (e.g., contractor unexpectedly hits bedrock). [See Restatement (Second) of Contracts §89] Under the majority view, however, mere unforeseen difficulty in performing is not a substitute for consideration. But if the unforeseen difficulty rises to the level of impracticability, such that the duty of performance would be discharged (see VI.E.5., infra), most states will hold that the unforeseen difficulty is an exception to the preexisting legal duty rule 6. Exception—Modification of Contract for the Sale of Goods At common law, a contract modification generally is unenforceable unless it is supported by new consideration. Article 2 does not follow this rule. Under Article 2, contract modifications sought in good faith are binding without consideration. A good faith modification is based on a legitimate commercial reason outside the control of the party seeking the modification. Modifications extorted from the other party are in bad faith and are unenforceable example: Paintco has agreed to sell to Retailco 15,000 gallons of paint at a price of $5 per gallon, to be delivered in 500-gallon installments each month for 30 months. After 15 months, the price of materials rises so that Paintco is losing 50¢ per gallon. Paintco had at the inception of the contract made a profit of 25¢ per gallon. Paintco tells Retailco the circumstances and asks if Retailco will agree to pay $5.75 per gallon for the remaining deliveries. Retailcoagrees and the proper writing is executed. The modification wasno doubt sought in good faith and is binding even though Paintco gave Retailco no new consideration. If Paintco had asked for an increased price because she believed that it was too late for Retailco to purchase elsewhere and Retailco would pay the higher price to get the paint, the modification would be in bad faith and would be unenforceable. 7. Existing Debts One of the recurring problems in the preexisting duty area concerns promises regarding existing debts. When the amount due is undisputed, payment of a smaller sum than due will not be sufficient consideration for a promise by the creditor to discharge the debt. Neither a legal detriment nor a benefit would be present. But again, bear in mind that courts will attempt to avoid this result by applica- tion of the above exceptions. Thus, for example, if the consideration is in any way new or different (e.g., payment before maturity or to one other than the creditor; payment in a different medium, e.g., stock instead of cash; or payment of a debt that is subject to an honest dispute), then sufficient consideration may be found.

Effect of illegality defense?

1. contract is void. 2. If the subject matter or consideration was illegal at the time of the offer, there was no valid offer. If it became illegal after the offer but before acceptance, the supervening illegality operates to revoke the offer. If it became illegal after a valid contract was formed, the supervening illegality operates to discharge the contract because perfor- mance has become impossible.

Statute of frauds rule under the UCC?

A contract for the sale of goods for a price of $500 or more is within the Statute of Frauds and generally must be evidenced by a signed writing to be enforceable. Note that a writing is sufficient even though it omits or incorrectly states a term, but the contract is not enforceable beyond the quantity of goods shown in the writing.

What is an exlclusitivity agreement?

A court may find an implied promise furnishing mutuality in appropriate circumstances (such as exclusive marketing agreements). The courts generally will find an implied promise to use best efforts and sustain agreements that might otherwise appear illusory.

is the merchant's confirmatory memo subject to the battle of the forms?

A merchant's memo confirming an oral agreement that contains different or additional terms is also subject to the battle of the forms provisions.

What is a misrepresentation?

A misrepresentation is a false assertion of fact. It is fraudulent if it is intended to induce a party to enter into a contract and the maker knows or believes the assertion is false or knows that he does not have a basis for what he states or implies with the assertion. A fraudulent assertion can be inferred from conduct; i.e., concealment or sometimes even nondisclosure may be considered a misrepresentation. If a party induces another to enter into a contract by using fraudulent misrepresentation (e.g., by asserting information she knows is untrue), the contract is voidable by the innocent party if she justifiably relied on the fraudulent misrepresentation. This is a type of fraud in the inducement. example: Buyer agreed to buy a painting from Seller because Seller told her that the painting previously had been owned by Bubbles Springfield, a famous rock star. In fact, Seller knew that Springfield had never owned the painting. Buyer's promise is voidable if she justifiably relied on Seller's misrepresentation.

What is a surety?

A suretyship contract involves a promise to pay the debt of another. A suretyship contract is not enforceable unless it is supported by consideration. If a surety is compensated, the requirement of consideration is not much of an issue, because the compensation will serve as consideration for the surety's promise. If, however, the surety is gratuitous (i.e., the surety is not paid for his services), the consideration requirement may cause problems. The timing becomes important in determining whether adequate consideration is present in a gratuitous surety situation.

What is disaffirmance?

An infant may choose to disaffirm a contract any time before (or shortly after) reaching the age of majority. If a minor chooses to disaffirm, she must return anything that she received under the contract that still remains at the time of disaffirmance. However, there is no obligation to return any part of the consider- ation that has been squandered, wasted, or negligently destroyed. exception - Necessaries - Necessaries" generally includes food, shelter, clothing, medical care, medicines, and other items necessary for the minor's subsistence, health, or education. A minor may disaffirm a contract for necessaries but in most states will be liable in restitution for the value of benefits received.

How is a mere inquiry different from a counteroffer?

An inquiry will not terminate the offer when it is consis- tent with the idea that the offeree is still keeping the original proposal under consideration. The test is whether a reasonable person would believe that the original offer had been rejected. Example: 1) The offeree says to the offeror, "Would you consider lowering your price by $5,000?" This, without more, is merely an inquiry, not a rejection. 2. 2) The offeree says to the offeror, "I couldn't possibly pay your asking price but could pay $5,000 less." This is more than a mere inquiry because of the certitude involved and will be treated as a counteroffer.

What is an offer?

An offer creates a power of acceptance in the offeree and a corresponding liability on the part of the offeror. For a communication to be an offer, it must create a reasonable expectation in the offeree that the offeror is willing to enter into a contract on the basis of the offered terms. In deciding whether a communication creates this reasonable expectation, you should ask the following three questions: (i) Was there an expression of a promise, undertaking, or commitment to enter into a contract? (ii) Were there certainty and definiteness in the essential terms? (iii) Was there communication of the above to the offeree?

How can beginning performance in response to a true unilateral offer limit the offeror's power to revoke?

An offer for a true unilateral contract becomes irrevocable once performance has begun. Note that the unilateral contract will not be formed until the total act is complete. However, once the offeree begins to perform, she is givena reasonable time to complete performance, during which time the offer is irrevocable. Note also that the offeree is not bound to complete perfor- mance—she may withdraw at any time prior to completion of performance. Example: Matt offers to pay Lisa $1,000 if she will paint his house, insisting that the acceptance occur only by the act of painting the house rather than through Lisa's promise. Lisa beginsto paint the house. Matt attempts to revoke the offer. Matt's attempt at revocation is ineffective because Lisa must have a reasonable time in which to complete the act of painting. If Matt refuses to allow Lisa to continue to paint, Matt will be in breach of contract and will be liable for damages.

What is timely acceptance under an option contract?

An offer must be accepted within the time specified or, if no time is specified, within a reasonable time. Often, an option contract specifies that the offer must be accepted within the option period; i.e., the offer terminates when the option expires. In the absence of specific language stating when the offer terminates, the power of acceptance arguably survives the option period, but courts often treat the option period as the offer period so that at the end of that time, the option expires and the offer lapses. Example: As in the above example, a farmer offers to sell her farm to a buyer for $1 million and promises to keep the offer open for 90 days if the buyer pays $1,000 to keep the offer open. The buyer pays the $1,000 but does not buy the farm during the 90-day period. On the 92nd day, the farmer sells the farm to someone else. Most courts would find that the option holder can no longer accept the offer, even if the farmer has not revoked it. The option period and offer period are treated as one and the same.

What is the requirment of multuality?

Consideration must exist on both sides of the contract; that is, promises must be mutually obliga- tory. There are many agreements in which one party has become bound but the other has not. Such agreements lack mutuality, i.e., at least one of the promises is "illusory." If so, consideration fails. Example: Acme Co. promises to buy from Batcher, Inc. "such ice cream as I may wish to order from Batcher, Inc." Acme's promise is illusory, because it is still free to buy from anyone else it chooses, or not to buy at all.

what is the article 2 rule for notice?

Article 2 has a slightly different rule regarding notice, although the end result is basically the same. It provides that when a contract is accepted by the beginning of performance, if the offeree fails to notify the offeror of the acceptance (i.e., the beginning of performance rather than the completion of performance) within a reasonable time, the offeror may treat the offer as having lapsed before accep- tance (i.e., no contract was ever formed, as opposed to the Restatement view that a contract was formed but performance is excused by failure of a condition).

Whats the note on the begginning performance?

As noted above, most offers are indifferent as to the manner of acceptance, and, thus, a bilateral contract may be formed upon the start of performance by the offeree. (See I.C.2.c.1), supra.) Therefore, once the offeree begins performance, the contract is complete and revocation becomes impossible. But note: Notification of the start of performance may be necessary. (See D.3.b., infra.)

When is a revocation effective?

At common law, a written communication is considered to have been "received" when it comes into the possession of the person addressed (orof someone authorized by him to receive it) or when it is deposited in some place authorized as the place for this or similar communications to be depos- ited. [Restatement (Second) of Contracts §68] The communication need not be read by the recipient to be effective. Similarly, under the UCC, a person receives notice when (i) it comes to his attention, or (ii) it is delivered at a place of business through which the contract was made or another location held out by that person as the place for receipt of such communications. An organization receives a communication at the time it is brought (or should have been brought) to the attention of the individual conducting the transac- tion. [UCC §1-202] Note that these rules do not restrict the communication or notice to a writing; thus, courts will likely apply the same rules to phone messages.

What is the effect of the offeree sending both acceptance and rejection?

Because a rejection is effective only when received, an offeree sending both an accep- tance and rejection could create problems for the offeror if the mailbox rule were appli- cable; e.g., a contract would be created when the acceptance was dispatched even if the offeror received the rejection and relied on it before receiving the acceptance. 1. Offeree Sends rejection, then acceptance - Mailbox rule does not apply - If the offeree sends a rejection and then sends an acceptance, the mailbox rule does not apply. Whichever one is received first is effective. Rejection sent first whatver gets received first is effective Offeree Sends Acceptance, Then Rejection—Mailbox Rule Generally Applies - If the offeree sends the acceptance first, the mailbox rule applies; i.e., a contract is created upon dispatch of the acceptance. However, if the offeror received the rejec- tion first and changed his position in reliance on it, the offeree will be estopped from enforcing the contract.

Rejection of an option contract?

Because an option is a contract to keep an offer open, a rejection of or a counteroffer to an option does not constitute a termination of the offer. The offeree is still free to accept the original offer within the option period unless the offeror has detrimentally relied on the offeree's rejection. [Restatement (Second) of Contracts §37]

What happens with acceptance in a unilateral contract?

COMPLETTION IS ACCEPTANCE BUT STARTING PERFORMANCE CREATES AN IRREVOCABLE OPTION CONTRACT Notice - Generally, the offeree is not required to give the offeror notice that he has begun the requested performance, but is required to notify the offeror within a reasonable time after performance has been completed. If a required notice is not given, a contract is formed, but the offeror's duties are discharged for failure of an implied condition subse- quent. However, no notice is required if: offeror waved notice or the offerees performance would normally come to the offeror's attention within a reasonable time. Example - Joe tells Susan he will pay her $1,000 if she paints the house that he is living in. Susan need not formally notify Joe that she has painted his house, as her performance would be obvious to him. Compare the distinction: In writing, Joe tells Susan that if she lends Tina $1,000 for one year, he will repay the loan if Tina fails to pay. Joe becomes contractually bound on his promise the instant Susan loans Tina the money, but Joe will be discharged from this contractual obligation unless Susan notifies him of her acceptance (i.e., her making the loan to Tina) within a reasonable time.

Are conditional promises enforceable?

Conditional promises are enforceable, no matter how remote the contingency, unless the "condition" is entirely within the promisor's control.

Defense of duress?

Contracts induced by duress (e.g., "sign the contract or I'll break your legs") or undue influ- ence are voidable and may be rescinded as long as not affirmed. There are two types of duress. In the first, a party is physically forced to sign against her will; e.g., a stronger person grabs her hand and signs the contract with the victim's example: Able tells Baker that Baker must sign a business contract with Able in which all provisions greatly favor Able. Able states that if Baker fails to sign, Able will hire someone to hurt Baker's teenage daughter. Baker signs the agreement. The contract is voidable at Baker's option.

How can offeree terminate an offer?

Express - basically saying nah i dont accept Counteroffer as rejection - A counteroffer is an offer made by the offeree to the offeror that contains the same subject matter as the original offer but differs in its terms. A counteroffer serves as a rejection of the original offer as well as a new offer. Examples: (i) Counteroffer combined with express rejection, e.g., "Not at that price, but I'll take it at $200." (ii) Acceptance conditional upon additional terms, e.g., "I'll take it at that price, but only if it is also equipped with air conditioning."

Does acceptance of a bilateral contract have to be communicated?

Generally, acceptance of an offer to enter into a bilateral contract must be communi- cated to the offeror. EXCEPTION: If an offer provides that acceptance need not be communicated, then no communi- cation of the acceptance is required. example: Alex applies for life insurance on a form that provides that the policy will become effective immediately upon approval by the insurance company's home office. The insurance contract is formed when the home office approves Alex's application.

What happens if a term is vague?

If a material term is vague or ambiguous, it is not an offer at common law or under the UCC. The presumption that the parties' intent was to include a reasonable term goes to supplying missing terms. However, the presumption cannot be made if the parties have included a term that makes the contract too vague to be enforced. The problem then is that the parties have manifested an intent that cannot be determined. Examples: 1) An agreement to divide profits "on a liberal basis" is too vague to be enforced. 2) An agreement to purchase a parcel of land for "$8,000 or less" is also too vague.

In a battle of the forms situation what happens when it is a contract between merchants?

If both parties to the contract are merchants, additional terms in the accep- tance will be included in the contract unless: 1. They materially alter the original terms of the offer (e.g., they change a party's risk or the remedies available); 2. The offer expressly limits acceptance to the terms of the offer. 3. The offeror has already objected to the particular terms, or objects within a reasonable time after notice of them is received. Example: Sellco emails an offer to sell to Buyco 1,500 widgets at $10 each. Buyco replies, "We accept, 5% discount for paying within 30 days." The parties have formed a contract and it probably includes a 5% discount for payment within 30 days (assuming that the discount is not material). See the distinction - Buyco emails Sellco an order for 1,500 widgets at $10 each. Sellco replies, "We accept. Warranties are limited to the express warranties in these attached documents. These warran- ties are in lieu of the implied warranty of merchantability." The parties have formed a contract. However, the warranty disclaimer is a material alteration that will not be included in the terms of the contract.

What is the illegality defense?

If either the consideration or the subject matter of a contract is illegal, this will serve as a defense to enforcement. Contracts may be illegal because they are inconsistent with the Constitution, violate a statute, or are against public policy as declared by the courts.

is past consideration any good?

If something was already given or performed before the promise was made, it will not satisfy the "bargain" requirement. The courts reason that it was not given in exchange for the promise when made. Exception - Debt Barred by a Technical Defense If a past obligation would be enforceable except for the fact that a technical defense to enforcement stands in the way (e.g., statute of limitations), the courts will enforce a new promise if it is in writing or has been partially performed. However, the court will enforce the contract only to the extent of the new promise. Promise to Pay Arising Out of Past Material Benefit—Material Benefit Rule Under a modern trend, some courts will enforce a promise if it is basedon a material benefit that was previously conferred by the promisee on the promisor and if the promisee did not intend to confer the benefit as a gift. This includes situations in which the promisee performed an act at the promi- sor's request or performed an unrequested act during an emergency (such as in the example in b.1), above). The Second Restatement follows this rule but adds that the promise is unenforceable to the extent it is disproportionate to the benefit conferred.

what is a material breach?

If the obligee does not receive the substantial benefit of her bargain as a result of failure to perform or defective performance, the breach is considered material. If the breach is material, the consequences are more severe. The nonbreaching party (i) may treat the contract as at an end, i.e., any duty of counterperformance owed by her will be discharged, and (ii) will have an immediate right to all remedies for breach of the entire contract, including total damages.

Can you cure uncertainty?

If uncertainty results because the offeree is given a choice of alternative performances, the offer becomes definite when the offeree communicates her choice. (See previous example about choice of motorcycle for $1,000.)

Remedies for fraud?

In addition to rescission, remedies for material misrepresentation or fraud includeall remedies available for breach (see VI., infra)

What is the statute of frauds?

In most instances, an oral contract is valid. However, certain agreements, by statute, must be evidenced by a writing signed by the party sought to be bound.

what are some common instances of procedural unconscionability?

Inconspicuous Risk-Shifting Provisions Standardized printed form contracts often contain a material provision that seeks to shift a risk normally borne by one party to the other. Examples of such provisions are: (i) Confession of judgment clauses, which are illegal in most states; (ii) Disclaimer of warranty provisions; and (iii) "Add-on" clauses that subject all of the property purchased from a seller to repos- session if a newly purchased item is not paid for. Typically, such clauses are found in the fine print ("boilerplate") in printed form contracts. Courts have invalidated these provisions because they are inconspicuous or incomprehensible to the average person, even if brought to his actual attention.

What is the minor's lack of capacity defense?

Infants generally lack capacity to enter into a contract binding on themselves. However, contractual promises of an adult made to an infant are binding on the adult. In other words, a contract entered into between an infant and an adult is voidable by the infant but binding on the adult.

act or forbearance by promisee must be of benefit to the promisor?

It is not enough that the promisee incurs detriment; the detriment must be the price of the exchange, and not merely fulfillment of certain conditions for making the gift. The test is whether the act or forbearance by the promisee would be of any benefit to the promisor. In other words, if the promisor's motive was to induce the detriment, it will be treated as consideration; if the motive was no more than to state a condition of a promise to make a gift, there is no consideration. example: "Come to my house and I will give you my old television." The promisee suffers a detriment by going to the promisor's house, as she did not have to go there at all. However, the promise of the television was probably not made to induce the promisee to come to the promisor's house. Hence, there is no consideration.

Legal detriment to promisee?

Legal detriment will result if the promisee does something he is under no legal obligation to do or refrains from doing something that he has a legal right to do. It is important to remember that the detriment to the promisee need not involve any actual loss to the promisee or benefit to the promisor example: Uncle promises Nephew $5,000 if he will refrain from drinking, smoking, swearing, and gambling until he reaches age 21. Nephew's refraining is a legal detriment, and because it was bargained for, Uncle must pay the $5,000 if Nephew so refrains. Note: Remember that the promisor must have sought to induce the detrimental act by his promise.

what is covered by the statute of frauds?

MY LEGS Marriage—when marriage is consideration for promise (e.g., "If you marry my son, I will buy you a car") Year—promises that cannot be performed within one year from date of contract Land—promises creating interests in land (e.g., leases, easements, fixtures, mineral rights, mortgages) Executors and administrators—promises to pay estate debts from own funds Goods—contracts for sale of goods for a price of $500 or more Executor or Administrator Promises Personally to Pay Estate Debts A promise by an executor or administrator to pay the estate's debts out of his own funds must be evidenced by a writing. Promises to Pay Debt of Another (Suretyship Promises) Promises in Consideration of Marriage Interest in Land - includes leases and easements of more than one year. not like a contract to build a building A promise that by its terms cannot be performed within one year is subject to the Statute of Frauds. Part performance does not satisfy the Statute of Frauds in this case, but if its possible to be completed in one year its not within the statut of frauds lifetime contracts are not within the statute of frauds.

Can acceptance be equivocal?

Mirror image rule - heavens no, must be unequivocal-- Traditional contract law insisted on an absolute and unequivocal acceptance of each and every term of the offer (the "mirror image rule"). At common law, any different or additional terms in the acceptance make the response a rejection and counteroffer.

Does preparation to perform limit the offerors power to revoke an offer?

No rule does not apply to preparations BUT substantial preparations me constitute detrimental reliance. Example: Matt offers to pay Lisa $1,000 if she will paint his house, insisting that the acceptance occur only by the act of painting. Lisa immediately drives to the local hardware store; expends $100 purchasing paint brushes, drop cloths, and masking tape to enable her to paint the house; and returns. On her return, Matt tells Lisa that he has changed his mind and does not want his house painted after all. Matt's revocation of his offer is valid because Lisa's acts did not constitute the beginning of perfor- mance, but rather were merely done in preparation to perform. However, Lisa will have an action against Matt to recover the $100 she spent in detrimental reliance on his offer.

In a Battle of the forms situaton do the new terms of offer govern if the contract has a nonmerchant?

No, If any party to the contract is not a merchant, the additional or different terms are considered to be mere proposals to modify the contract that do not become part of the contract unless the offeror expressly agrees. example: Paul sends a letter offering to sell his car to Stephanie for $1,200. Stephanie sends Paul a letter stating: "I accept and want you to put new tires on it." This is a contract, but Paul is not bound to put new tires on the car.

What defense does an intoxicated person have?

One who is so intoxicated that he does not understand the nature and significance ofhis promise may be held to have made only a voidable promise if the other party had reason to know of the intoxication. The intoxicated person may affirm the contract upon recovery. Once again, there may be quasi-contractual recovery for necessaries furnished during the period of incapacity.

What is the mental incapacity defense?

One whose mental capacity is so deficient that he is incapable of understanding the nature and significance of a contract may disaffirm when lucid or by a later appointed legal representative. He may likewise affirm during a lucid interval or upon complete recovery, even without formal restoration by judicial action. In other words, a contract made by a mentally incompetent person is voidable. As in the case of infants, mentally incompetent persons are liable in quasi-contract for necessaries furnished to them.

What are some limations on an offeror's power to revoke?

Option contracts - An option is a distinct contract in which the offeree gives consideration for a promise by the offeror not to revoke an outstanding offer.

What about acceptance of offer for a bilateral contract?

Recall that unless an offer specifically provides that it may be accepted only through perfor- mance, it will be construed as an offer to enter into a bilateral contract and may be accepted either by a promise to perform or by the beginning of performance (compare offers for true unilateral contracts, which may be accepted only by full performance).

How can an offeror terminate an offer?

Revocation - this can be direct Offers made by publication may be terminated by publication of revoca- tion through comparable means. Example: An offer published in The New York Times may be revoked by publication in The New York Times. It may not be revoked by publication in Reader's Digest or by a TV spot. Offer can also be revoked by indirect communication if: (1) the offeree indirectly receives correct information, from a reliable soure, (3) of acts of the offeror that would indicate to a reasonable person that the offeror no longer wishes to make the offer. Example: Offeree, before attempting to accept Offeror's offer to sell Greenacre, was informed by a reliable third party that Offeror had sold Greenacre to another. Held: Offeror revoked the offer.

What is a bilateral contract formed by performance?

Sometimes in business, a contract is not formed by the parties' communications, either because: (i) the mirror image rule has not been satisfied; or (ii) in a contract for the sale of goods, the original offeror's form contains a clause objecting in advance to any new or incon- sistent term and the offeree sends a response with new or different terms that states it is not an acceptance unless the original offeror agrees to these terms. Clearly, no contract is formed at this point. But, as is sometimes the case, if the parties begin to perform as if they formed a contract, a contract is formed. Rationale: At common law, the last communication sent to the party who performed is considered a counteroffer and the performance is considered accep- tance of the counteroffer. In contracts for the sale of goods, Article 2 specifically provides that conduct by both parties that recognizes the existence of a contract is sufficient to estab- lish the contract.

writing requirement under the statute of frauds?

The Statute requires only one or more writings, signed by the party to be charged, that: (i) reasonably identify the subject matter of the contract, (ii) indicate that a contract has been made between the parties, and (iii) state with reasonable certainty the essential terms of the unperformed promises.

What does definite and certain terms mean?

The basic inquiry is whether enough of the essential terms have been provided so that a contract including them would be capable of being enforced. The principle is that the parties make their own contract; the courts do not make it for them. What is essential for the requisite certainty in an offer depends on the kind of contract contemplated. Typically, the following are important: (i) the identity of the offeree; (ii) the subject matter; and (iii) the price to be paid. However, a promise generally will be enforceable even if it does not spell out every material term, as long as it contains some objective standard for the court to use to supply the missing terms. identity must be sufficient to identify the offeree or the class they belong too.

How does the method of communication figure in?

The broader the communicating media (e.g., publications), the more likely it is that the courts will view the communication as merely the solicitation of an offer. (Note that there is an exception as to reward offers.) Advertisements - Advertisements, catalogs, circular letters, and the like containing price quota- tions are usually construed as mere invitations for offers. They are announce- ments of prices at which the seller is willing to receive offers. Typically, these are not considered offers because they usually are indefinite as to quantity and other terms, and addressed to the general public. If an advertisement addressed to the general public were considered an offer, it might be overaccepted; i.e., the number of acceptances may exceed the number of items for sale.

Unconscionability

The concept of unconscionability allows a court to refuse to enforce a provision or an entire contract (or to modify the contract) to avoid "unfair" terms. It is sometimes said that there are two types of unconscionability: substantive unconscionability (i.e., unconscionability based on price alone) and procedural unconscionability (i.e., unconscionability based on unfair surprise or unequal bargaining power). However, few cases recognize substantive unconscionability based on unfair price alone. Instead, the cases have dealt mostly with procedural unconscionability.

must the offeree know of the offer?

The offeree must know of the offer in order to accept, and this is true whether the offer is for a bilateral or unilateral contract. Examples: 1) Alex sends Becky a letter offering to sell her Blackacre for $500,000. That same day, without knowledge of Alex's letter, Becky sends Alex a letter offering to buy Blackacre from him for $500,000. No contract is formed because neither party knew of the other's letter when sending his or her own letter. 1) Alex sends Becky a letter offering to sell her Blackacre for $500,000. That same day, without knowledge of Alex's letter, Becky sends Alex a letter offering to buy Blackacre from him for $500,000. No contract is formed because neither party knew of the other's letter when sending his or her own letter. 2) Cindy offers to pay $1,000 to the person who finds her missing dog. Dee finds a dog in her yard, reads its tags, and returns the dog to Cindy without knowledge of Cindy's offer. Most courts hold that no contract is formed here.

What is forbearance to sue?

The promise to refrain from suing on a claim may constitute consideration. If the claim is valid, the forbearance to sue is, of course, sufficient consideration. If the claim is invalid and the claimant is aware of this fact, he has no such right; his suit is no more than the wrongful exercise of a power. But even if the claim is invalid, in law or in fact, if the claimant reasonably and in good faith believes his claim to be valid, forbearance of the legal right to have his claim adjudicated constitutes detriment and consideration.

What is the rule for the shipment of nonconforming goods?

The shipment of nonconforming goods is an acceptance creating a bilateral contract as well as a breach of the contract unless the seller seasonably notifies the buyer that a shipment of nonconforming goods is offered only as an accommoda- tion. The buyer is not required to accept accommodation goods and may reject them. If he does, the shipper is not in breach and may reclaim the accommodation goods, because her tender does not constitute an acceptance of the buyer's original Examples: 1. 1) Craig orders 1,500 blue widgets from Susy. Susy ships 1,500 black widgets but does not notify Craig that the goods are offered only as an accommodation. Susy's shipment is both an acceptance of Craig's offer and a breach of the resulting contract. Craig may sue for any appropriate damages. 2. 2) In the example above, Susy, before the goods arrive, notifies Craig that black widgets have been sent as an accommodation. The shipment is a counteroffer and, if Craig accepts delivery, there will be a contract for the purchase of black widgets. 3. 3) Craig orders 1,500 blue widgets from Susy. Susy sends Craig an email promising to ship the widgets within two days. Upon checking her stock, Susy discovers that she has only 1,000 blue widgets. She ships the 1,000 blue widgets along with 500 black widgets and a letter explaining that the black widgets are offered only as an accommodation. Craig may sue for damages. Susy accepted Craig's order via email, promising to ship 1,500 blue widgets. This is not a case of acceptance by shipment.

How does the definitiveness of subject matter factor into an offer?

The subject matter of the deal must be certain, because a court can enforce a promise only if it can tell with reasonable accuracy what the promise is. For real estate contracts - land and price terms required. for sale of goods, quantity must be required. except in requirements and output contracts

what is a bilateral contract accepted?

The traditional bilateral contract is one consisting of the exchange of mutual promises, i.e., a promise for a promise, in which each party is both a promisor and a promisee.

what is a unilateral contract?

The traditional unilateral contract is one in which the offeror requests performance rather than a promise. Here, the offeror-promisor promises to pay upon the completion of the requested act by the promisee. Once the act is completed, a contract is formed. In such contracts, there is one promisor and one promisee. example: Susan promises to pay Charles $5 if he will deliver a textbook to Rick. Charles is not obligated to deliver the book, but if he does in fact deliver it, Susan is obligated to pay him the $5

May unilateral misake be canceled in equity?

There is authority in a number of cases that contracts with errors, such as mistakes in computation, may be canceled in equity, assuming that the nonmistaken party has not relied on the contract. There is also modern authority indicating that a unilateral mistake that is so extreme that it outweighs the other party's expectations under the agreement will be a ground for cancellation of the contract.

What does communication to the offeree mean?

To have the power to accept, the offeree must have knowledge of the offer. Therefore, the proposal must be communicated to her. Example: Chauncey returned Bowater's lost briefcase unaware that Bowater had placed an advertisement offering a $20 reward for its return. Because the offer had not been communicated to Chauncey, there could not be mutual assent. Hence, there is no contract.

Preexisting duty rule?

Traditionally the promise to perform, or the performance of, an existing legal duty is not consideration. examples: 1) Mike contracts to build a garage for Richard for $15,000. Mike discovers that he cannot make a profit at that price and tells Richard that he will not build the garage unless Richard promises to pay him $16,000. Because Richard does not have time to find a new contractor before winter and he does not want his new car exposed to snow, he agrees to pay Mike the $16,000. When Mike finishes the garage, Richard pays Mike $15,000. Mike cannot enforce the promise for the additional $1,000 because he was under a preexisting duty to build the garage. 2. 2) Smith offers a $10,000 reward for recovery of his kidnapped daughter. Jones, a police officer assigned to this case, recovers the daughter. Jones's performance of her official duty is not sufficient consideration.

How can lapse of time reject an offer?

Two things to know: 1. Must Accept Within Specified or Reasonable Time - The offeree must accept the offer within the time period specified or, if no time period is specified, within a reasonable time. If she does not do so, then she will have allowed the offer to terminate. (Note: Where the offer's terms are unclear as to time, e.g., "by return mail," the time limit is what a reason- able person in the offeree's position would have assumed.) 2. Look to when offer is received by offeree - If the offer provides that it will expire within a particular time period, that period commences when the offer is received by the offeree. If the offer is delayed in transmission and this fact is or should have been apparent to the offeree, the offer terminates at the time it would have expired had there been no delay. All relevant facts must be considered in determining whether this knowledge is present. These include, e.g., date of letter, postmark, and any subsequent statements made by the offeror.

What is the merchant's firm offer under article 2?

Under Article 2, there are circumstances in which a promise to keep an offer open is enforceable even if no consideration has been paid to keep the offer open. Under Article 2: (1) if a merchant (2) offers to buy or sell goods in a signed writing; and (3) the writing gives assurances that it will be held open; (4) the offer is not revocable for lack of consideration during the time stated, or if no time is stated for a reasonable time BUT CANNOT EXCEED 3 months.

What is a unilateral mistake?

Unilateral mistakes arise most commonly when one party makes a mechanical error in computation. If only one of the parties is mistaken about facts relating to the agreement, the mistake will not prevent formation of a contract. However, if the nonmistaken party knew or had reason to know of the mistake made by the other party, the contract is voidable by the mistaken party. As is the case with mutual mistake, for the contract to be voidable, the mistake must have a material effect on the agreed upon exchange and the mistaken party must not have borne the risk of the mistake. Materiality is determined by the overall impact on both parties. Ordinarily this is proven by showing the exchange is much less desirable to the mistaken party and more advantageous to the nonmistaken party. example: Seller agrees to sell Buyer a number of different items of hardware. Seller computes the total price at $15,000, and Buyer agrees to pay this amount. Subsequently, Seller discovers that he made an error in computation and the price should be $17,000. In this situation, the preferred analysis is that there is a contract at $15,000, assuming that Buyer was reasonably unaware of the unilateral computation error. Note also that the error was not an error in the offer; the mistake was antecedent to the offer by Seller. When Seller stated the offer at $15,000, he meant $15,000. Compare the distinction - Homeowner asks four contractors to submit bids to build a two-car garage on Homeowner's property. When Homeowner receives the bids they are: $17,000, $19,000, $19,500, and $9,000. The last bid was due to a typograph- ical error. Homeowner will not be able to snap up the $9,000 offer because he should have known, based on the other bids, that the $9,000 bid probably contained an error.

What are the three questions to determine if there was in fact a contract?

Was there mutual assent? Was there consideration or some substitute for consideration? Are there any defenses to creation of the contract?

what is the effect of conditional acceptance?

When an acceptance is made expressly conditional on the acceptance of new terms, it is a rejection of the offer. The conditional acceptance is essentially a new offer, and the original offeror may form a contract by expressly assenting to the new terms. However, the offer that results from a conditional acceptance cannot be accepted by performance. If the parties ship or accept goods after a condi- tional acceptance, a contract is formed by their conduct, and the new terms are not included. The contract consists of all terms on which their writings agree, plus supplementary terms supplied by the UCC. Example: Same facts as the example in b.1), supra, except that after Sally's e-mail making her acceptance conditional on Harry's consent to the delivery term, Harry does not reply but delivers the car to Sally's house two days later. Sally accepts the delivery. Harry is not in breach because Sally's conditional acceptance was not a counter- offer that could be accepted by performance. When Sally condi- tioned her acceptance on consent to the delivery term, there was no contract. The contract was not formed until Harry delivered the car and Sally accepted it. The contract was formed by performance and its terms are those to which the parties agreed (i.e., $1,000 for the car) plus the terms supplied by their course of performance (delivery two days later at Sally's house) and the UCC.

When is a writing not required under the UCC?

When it is specially manufactured goods - If goods are to be specially manufactured for the buyer and are not suitable for sale to others by the seller in the ordinary course of his business, the contract is enforceable if the seller has, under circumstances that reasonably indicate that the goods are for the buyer, made a substantial beginning in their manufacture or commitments for their purchase before notice of repudiation is received. admissions in a pleadings or court - f the party against whom enforcement is sought admits in pleadings, testi- mony, or otherwise in court that the contract for sale was made, the contract is enforceable without a writing (but in such a case the contract is not enforced beyond the quantity of goods admitted) payment or delivery of goods - If goods are either received and accepted or paid for, the contract is enforce- able. However, the contract is not enforceable beyond the quantity of goods accepted or paid for. Thus, if only some of the goods called for in the oral contract are accepted or paid for, the contract is only partially enforceable. If an indivisible item is partially paid for, most courts hold that the Statute of Frauds is satisfied for the whole item. Examples: 1) Ketty and Lydia orally agree that Lydia will purchase 150 widgets from Ketty at a price of $10 each. Lydia gives Ketty a check for $70. The contract is enforceable for seven widgets only. 2) Joe orally contracts to buy a car from Suzette for $15,000. Joe gives her a $1,000 down payment. Although Joe has only partially paid for the car, most courts would hold that the contract is enforceable.

How does detrimental reliance limit the power to revoke?

Where the offeror could reasonably expect that the offeree would rely to her detriment on the offer, and the offeree does so rely, the offer will be held irrevocable as an option contract for a reasonable length of time. [Restatement (Second) of Contracts §87] The case law indicates that this may be limited to those situations in which the offeror would reasonably contem- plate reliance by the offeree in using the offer before it is accepted. Exception: A general contractor solicited bids from various subcontrac- tors before making its own irrevocable offer on a construc- tion project. For the subcontractor to be held to its offer, the subcontractor must reasonably have foreseen the possible use of its subcontracting bid in the making of the general contrac- tor's irrevocable offer.

Can you cure vague terms?

Yes, with part performance. BUT offeree must know of the offer in order to accept.

What must be the method of acceptance?

any reasonable manner and by any medium reasonable under the circumstances. Example: Nikki telephones an offer to Skip that is to remain open for five days.Two days later, Skip e-mails an acceptance, or two days later Skip mails an acceptance. Whether there has been a proper acceptance depends on whether the use of e-mail or mail was reasonable under the circumstances.

How may offers to buy goods for current or prompt shipment be accepted?

by either a promise to ship or by a shipment of conforming or nonconforming goods.

what determines if the communication is an offer?

can use language like i offer or i promise but its not necessary. Also, certain language is generally construed as merely contemplating an invitation to deal, preliminary negotiations, or "feelers," rather than being an offer. This includes phrases such as "I quote," "I am asking $30 for," and "I would consider selling for." No mechanical formula is available. For example, price quotes, which are gener- ally considered invitations to deal, can be offers if made in response to an inquiry that contains a quantity term. Courts will consider surrounding circumstances - The circumstances surrounding the language will be considered by courts in deter- mining whether an offer exists. For example, where the statement is made in jest, anger, or by way of bragging, and the statement is reasonably understood in this context, it will have no legal effect. However, where the statement is subjectively intended to be in jest but reasonably understood by the hearer to have been made seriously, the statement is an offer because it is interpreted objectively (i.e., according to a reasonable person's expectations). prior practice may determine if there was an offer too.

What is the knockout rule?

conflicting terms in the offer and accep- tance are knocked out of the contract because each party is assumed to object to the inclusion of such terms in the contract. Under the knockout rule, gaps left by knocked-out terms are filled by the UCC (e.g., when the date of delivery differs in the offer and the acceptance, the UCC provides that delivery must be made within a reasonable time). Even though it is possible for a contract to be formed despite the fact that the acceptance has different terms, there must still be a meeting of the minds. Differences between the offer and acceptance as to price, quantity, or quality indicate there is no meeting of the minds, and thus there is no contract. Example: Sellco offers to sell to Buyco 1,500 widgets at $10 each plus freight. Buyco replies, "I accept. The price is $10.10 each including freight." There is a contract, assuming that the actual cost of freight is not materially different from $150, as the offer was to sell the widgets for $10 plus (actual) freight costs and the acceptance was to purchase the widgets at $10.10 each including freight costs (the extra 10¢ apparently being to cover freight costs). Under the approach treating different terms like additional terms, the buyer's acceptance will control (i.e., the buyer is obligated to pay $15,150 for the widgets and no additional freight charges) unless the seller objects. If the seller does object, there is a contract on the seller's original terms. However, under the knockout rule, the different freight terms will be knocked out. The price will be $10 plus the reasonable cost of freight at the time of delivery according to the UCC.

What is termination by operation of law?

death or insanity, destruction of subject matter.

is economic duress a defense?

generally no aking advantage of another person's economic needs is not a defense. However, withholding something someone wants or needs will constitute economic duress if: (i) The party threatens to commit a wrongful act that would seriously threaten the other contracting party's property or finances; and (ii) There are no adequate means available to prevent the threatened loss. example: Barry buys his dream home for $700,000. A few years later, Barry loses his job, stops making mortgage payments, and is threatened with foreclosure. Because of the economy, houses are not sellingin Barry's neighborhood, so his friend Freida offers to buy Barry's house for the $500,000 that he owes on it. Before the closing, Barry finds a job and does not want to sell the house. Barry is bound; his economic duress is not a defense.

What happens if the price terms are missing?

he fact that one or more terms are left open does not prevent the formationof a contract if it appears the parties intended to make a contract and there isa reasonably certain basis for giving a remedy. In such a case, the majority of jurisdictions and Article 2 hold that the court can supply reasonable terms for those that are missing. [See UCC §§2-204, 2-305] These terms will be supplied, however, only where they are consistent with the parties' intent as otherwise expressed. Note that the more terms the parties leave open, the less likely it is that they intended to enter into a binding agreement. Article 2 gap filler - Article 2 includes some very specific "gap fillers" for situations where certain terms are not included in a contract for the sale of goods. Under Article 2, the price will be a reasonable price at the time of delivery if: 1. nothing is said to price 2. the price is left to be agreed to by the parties and they fail to agree 3. the price is fixed by some external factor or third party and it is not so set.

What is the perfect tender rule?

he perfect tender rule—if goods or their delivery fail to conform to the contract in any way, the buyer generally may reject all, accept all, or accept any commercial units and reject the rest.

What if time is left out?

if agreement does not specify time courts will make it a reasonable time.

What is a mutual mistake?

if both parties were mistaken as to a material fact at the time of contracting, either party can rescind the agreement. This material fact can be the existence, identity, or character of the subject matter. Does not apply when parties are doubtful of a fact but enter the contract anyway.

When may an ad be an offer?

in certain situa- tions, courts have treated advertisements as offers if the language of the advertise- ment can be construed as containing a promise, the terms are certain and definite, and the offeree(s) is clearly identified. Price quotations also may be considered offers if given in response to an inquiry. example: Defendant store advertised a particular coat worth $140 for $1 on a "first come, first served" basis. Held: Valid offer to first person accepting on this basis as nothing was left open for negotiation.

When can contract be removed from statute of frauds?

in land sale contracts - If a seller conveys to the buyer (i.e., fully performs), he can enforce the buyer's oral promise to pay. Likewise, the buyer may seek to specifically enforce an oral land sale contract under the doctrine of part performance. Part performance that unequivocally indicates that the parties have contracted for the sale of land takes the contract out of the Statute of Frauds. What constitutes sufficient part perfor- mance varies among jurisdictions. Most require at least two of the following: (i) payment (in whole or in part), (ii) possession, and/or (iii) valuable improvements. sale of goods contracts - Part performance is sufficient to take a sale of goods contract out of the Statuteof Frauds when: (i) the goods have been specially manufactured, or (ii) the goods have been either paid for or accepted. If a sales contract is only partially paid for or accepted, the contract is enforceable only to the extent of the partial payment or acceptance.

Will an acceptance by unauthorized means be ok?

maybe, An acceptance transmitted by unauthorized means or improperly transmitted by autho- rized means may still be effective if it is actually received by the offeror while the offer is still in existence. examples: 1. 1) Bailey makes an offer to Janet specifying that acceptance should be by email. Janet mails Bailey her acceptance. The acceptance will not be effective upon dispatch of the letter but only upon receipt by Bailey, if the offer is still open. 2. 2) Janet, in a situation where the mailbox rule otherwise applies, incor- rectly addressed the envelope in mailing back the acceptance. It will be effective upon receipt if the offer is still open.

what again is the merchant's confirmatory memo?

n contracts between merchants, if one party, within a reasonable time after an oral agreement has been made, sends to the other party a written confirmation of the understanding that is sufficient under the Statute of Frauds to bind the sender,it will also bind the recipient if: (i) he has reason to know of the confirmation's contents; and (ii) he does not object to it in writing within 10 days of receipt.

is a request for clarification a counteroffer?

no

IS mutual mistake a defense if party bore the risk?

no Mutual mistake is not a defense if the party asserting mistake as a defense bore the risk that the assumption was mistaken. This commonly occurs when one party is in a position to better know the risks than the other party (e.g., contractor vs. homeowner) or where the parties knew that their assumption was doubtful (i.e., when the parties were consciously aware of their ignorance). In other words, to be a defense it must be a mistake, not a mere uncertainty. Examples: 1. 1) Homeowner contacts Builder regarding the cost of installing an inground pool. Builder bids $15,000 and Homeowner accepts. While digging the hole for the pool, Builder encounters an unexpected slab of granite. Blasting away the granite will add 20% to Builder's costs, making the contract unprofitable. Builder will be held to have assumed the risk 2. 2) Roger finds a stone that appears to be valuable and shows it to his friend Betsy. The two do not know what the stone is but think it is a topaz. Roger agrees to sell the stone to Betsy for $100. The parties subsequently discover that the stone is a diamond worth $1,000. Roger cannot void the contract on mutual mistake grounds because the parties knew that their assumption about the stone was doubtful. Compare: Roger finds a stone that appears to be valuable. Because Roger is not an expert as to gems, he takes it to Jeweler. Jeweler, in good faith, tells Roger that the stone is a topaz worth very little and offers to buy it for $100. Roger accepts, but subsequently discovers that the stone actually is a diamond worth $1,000. Roger can rescind the contract on mutual mistake grounds. Roger's reliance on an expert's opinion shows that Roger did not intend to assume the risk of not knowing about the stone. Compare: Roger finds a stone that appears to be valuable. Because Roger is not an expert as to gems, he takes it to Jeweler. Jeweler, in good faith, tells Roger that the stone is a topaz worth very little and offers to buy it for $100. Roger accepts, but subsequently discovers that the stone actually is a diamond worth $1,000. Roger can rescind the contract on mutual mistake grounds. Roger's reliance on an expert's opinion shows that Roger did not intend to assume the risk of not knowing about the stone.

is mistake in value a defense?

no unless facts show that the adversely effected party did not assume the risk.

What is mutual assent?

offer and acceptance

What about an act as acceptance?

offeror can require that. Example: Jennifer offers to purchase Steve's car for $1,000, specifying that Steve accept the offer by wearing a yellow shirt to lunch next Tuesday. Steve can accept the offer only by acting as requested. If Steve simply tenders the automobile, most courts would construe the tender as a rejection and counteroffer. Also, recall that Steve must know of the offer to accept. If he simply wears a yellow shirt without knowing of Jennifer's offer, there is no acceptance and no bilateral contract.

When is acceptance effective?

the mailbox rule - Acceptance by mail or similar means creates a contract at the moment of dispatch, provided that the mail is properly addressed and stamped, unless: 1. offer stipulates that acceptance is not effective until received or 2. an option contract is involved - options can only be accepted upon recepipt

Who can accept?

the person the acceptance is addressed to.

Battle of the Forms (UCC)

the proposal of additional or different terms by the offeree in a definite and timely acceptance does not constitute a rejection and counteroffer, but rather is effective as an acceptance, unless the acceptance is expressly made conditional on assent to the additional or different terms. Example: Harry sends Sally an e-mail offering to sell her his car for $1,000. Sally e-mails back, "I accept; deliver it to my house by noon tomorrow." At common law, no contract would be formed here because Sally's acceptance added a delivery term. Under Article 2, a contract is formed and whether or not Harry is required to deliver the car to Sally's house by noon of the next day is determined by the rules discussed below. compare: Same facts as above, but Sally's e-mail says, "I accept, but only if you agree to deliver the car to my house by noon tomorrow." No contract is formed here because Sally's acceptance was expressly conditioned on assent to the new terms.

what if neither party was aware of the ambiguity?

there is no contract. If neither party was aware of the ambiguity at the time of contracting there is no contract unless both parties happened to intend the same meaning. example: Buyer agrees to purchase cotton from Seller when the cotton is delivered by a ship named Peerless. This is the total expression of the agreement. It is subsequently determined that Buyer contemplated a ship named Peerless that was to dock in September while Seller contemplated a ship named Peerless that was to dock in December. Neither party was aware that there were two ships named Peerless. Their subsequent expression of the ship each intended indicates that they did not intend the same ship at the time of contracting. Therefore, there is no contract.

what happens when on party is aware of the ambiguity?

theres a contract example: Collector agrees to purchase a Picasso sketch from Gallery. It is subse- quently determined that Gallery has two sketches and that Gallery intended to sell one of these to Collector while Collector intended to buy the other one. Collector did not know that Gallery owned two sketches; Gallery, of course, knew that it did. Here, there is a contract for the sketch that Collector had in mind because this is a situation in which one party knew of the ambiguity (Gallery) while the other party did not (Collector).

Does a grumbling acceptance constitute acceptance?

yes as long as it stops short of dissent


Related study sets

FIN3403- Chapter 5: Q.1, 2, 3, 4, 7 and Key Terms

View Set

CSP 031A C++ Programming I Midterm

View Set

Slope Formula, Missing Coordinate, Zeros

View Set

Lesson 10: Combining SAS Data Sets

View Set